You are on page 1of 20

Advanced Calculus Chapter 3 Summary 2019-2020

Sequences
Definition A sequence {pn } in X is a function f : N → X such that f (n) = pn ∈ X.

Definition The sequence {pn } converges in X if there exists p ∈ X such that for each  > 0,
there exists N ∈ N such that if n > N then d(p, pn ) < . We say {pn } converges to p. Or
equivalently, p is the limit of {pn }.

Notation pn → p ⇐⇒ lim {pn } = p.


n→∞

Definitions
(a) If {pn } does not converge to any point in X then it diverges.
(b) The range of {pn } is {x ∈ X | x = pn for some n}.
(c) We say {pn } is bounded in X if the range of {pn } is bounded in X.

True/False Questions Let {pn } be a sequence in a metric space X.


(a) If pn → p and pn → p0 , then p = p0 . True.
(b) If {pn } is bounded, then {pn } converges. False.
(c) If pn converges, then {pn } is bounded. True.
(d) If pn → p, then p is a limit point of the range of {pn }. False.
(e) If p is a limit point of E ⊆ X, then there exists some sequence {pn } such that pn → p. True.
(f) pn → p if and only if every neighborhood of p contains all but finitely many terms in {pn }.
True.
Remark “All but finitely many” is equivalent to “Almost all.”
Solution

a) Let  > 0. Then there exists N1 ∈ N such that n > N1 , d(p, pn ) < . Similarly there exists
2
0 
N2 ∈ N such that n > N2 , d(p , pn ) < . Then for n > max (N1 , N2 ):
2
2
d(p, p0 ) ≤ d(p, pn ) + d(p0 , pn ) < = .
2
Thus d(p, p0 ) <  for all  > 0. Thus d(p, p0 ) = 0. Hence p = p0 .

b) Consider Ex. (4) where there is oscillation between two points.

c) Suppose pn → p. Since 1 > 0 (our choice of ), there exists N ∈ N such that for n > N
implies d(pn , p) < 1. Let r = max (1, d(p, p1 ), . . . , d(p, pN )) + 1. Then pn ∈ B(p, r) for all n ∈ N.

d) Consider Ex. (3) where the sequence is simply one point.

e) p ∈ E 0 . For all n ∈ N such that pn ∈ E such that d(pn , p) < n1 and pn 6= p. There for a
1 1 1
sequence {pn }. Let  > 0. Choose N > . Then for each n > N , we have d(p, pn ) < < < .
 n N
Advanced Calculus Chapter 3 Summary(Continued) 2019-2020

Thus this sequences {pn } converges to p as desired.

f) (⇒) Suppose pn → p. Given B(p, ), there exists N ∈ N such that when n > N , it fol-
lows that d(p, pn ) < , i.e. pn ∈ B(p, ), leaving only finitely many points, p1 through pn possible.

(⇐) For all  > 0, B(p, ) contains almost all {pn }. For  > 0, let

m = max{n ∈ N | pn 6∈ B(p, )}.

Then n > m implies pn ∈ B(p, ), i.e. d(pn , p) < .

Theorem (Limit Laws) Suppose {sn }, {tn } are sequences in C, and lim sn = s, lim tn = t.
n→∞ n→∞
Then
(a) lim (sn + tn ) = s + t.
n→∞
(b) lim (csn ) = cs for all c ∈ C.
n→∞
(c) lim (c + sn ) = c + s for all c ∈ C.
n→∞
(d) lim (sn tn ) = st.
n→∞
1 1
(e) lim = provided sn 6= 0 for all n ∈ N, and s 6= 0.
n→∞ sn s
Outline of the Proof
(a) Let  > 0. Since lim sn = s and lim tn = t, there exists N1 , N2 ∈ N such that if n ≥ N1 , and
n→∞ n→∞
 
if n ≥ N2 then |sn − s| < and |tn − s| < , respectively. This imples that if n ≥ max (N1 , N2 ),
2 2
then

|(sn + tn ) − (s + t)| = |(sn + tn ) − (s + t)|


= |sn − s| + |tn − t|
< .

(b) Let  > 0. Since lim sn = s, there exists N ∈ N such that if n ≥ N, then |sn − s| < .
n→∞ 1 + |c|
|c| 
This imples that if n ≥ N, then |csn − cs| ≤ |c| |sn − s| < < .
1 + |c|

(d) Let  > 0. Choose k = max(|s|, |t|, 1, ). Since lim sn = s and lim tn = t, there exists
n→∞ n→∞
 
N1 , N2 ∈ N such that if n ≥ N1 , and if n ≥ N2 then |sn − s| < and |tn − s| < , respectively.
3k 3k
This imples that if n ≥ max (N1 , N2 ), then

|sn tn − st| = |(sn − s)(tn − t) + s(tn − t) + t(sn − s)|


2 |s| |t|
≤ 2+ +
9k 3k 3k
   |s| |t| 
≤ + + since k ≥ |s|, |t|,  =⇒ 1 ≥ , ,
9k 3 3 k k k
  
≤ + + since k ≥ 1
9 3 3
< .

Page 2
Advanced Calculus Chapter 3 Summary(Continued) 2019-2020

(e) Let  > 0. Since lim sn = s 6= 0, there exists N1 , N2 ∈ N such that


n→∞

1 1 1
if n ≥ N1 , then |sn − s| < |s| =⇒ |s| − |sn | ≤ |sn − s| < |s| =⇒ |s| ≤ |sn |
2 2 2
1 2
if n ≥ N2 , then |sn − s| < |s| , respectively.
2
This imples that if n ≥ max (N1 , N2 ), then

1 1 sn − s
− =
sn s sn s
2
< 2 |sn − s|
|s|
< .

Subsequences
Definition Let {pn } be a sequence in X and let {ni } be a sequence of natural numbers such
that n1 < n2 < n3 < . . . . Then {pni } is a subsequence of {pn }.

Examples
(a) Let {pn } = {1, π, 21 , π, 31 , π, . . . }. Notice this does not converge. But a subsequence does
converge. For example, only the π terms. As do the other terms.
(b) Notice that { 12 , 23 , 34 , 54 , . . . } converges for 1. Notice that any subsequence converges. By
property (f ) on Page 1, almost all terms of the subsequence are contained in any neighbor-
hood of the limit.

Remarks
(a) Note that if pn → p and {pnk } is any subsequence of {pn }, then pnk → p.
(b) Question: Must every sequence contain a convergent subsequence?
No. Consider the sequence {1, 2, 3, . . . }, and note that there is no convergent subsequence
of this sequence.
Recall that an ordered field F is said to have the least-upper-bound property if every nonempty,
bounded above subset E of F , the sup E ∈ E 0 exists in F. For general metric spaces without the
least-upper-bound property, the following generalizations hold.

Theorem
(a) In a compact metric space X, every sequence contains a subsequence converging to a point
in X, i.e. if {pn } is a sequence in a compact metric space, then there exists a subsequence
{pnj } of {pn } converges to a point p ∈ X.
Proof The proof is simmilar to the proof of Theorem 2.41 (b) =⇒ (c) on p40.
(b) (Bolzano-Weierstrass) Every bounded sequence in Rk contains a convergent subsequence.

Proof Let R = {pn | n ∈ N} = the range of {pn }.

Page 3
Advanced Calculus Chapter 3 Summary(Continued) 2019-2020

Suppose R is a finite subset of Rk , there exists a point p ∈ R and a subsequence {nj | j ∈


N} of {n | n ∈ N} such that
pnj = p ∀ j ∈ N
by the pigeon-hole principle. Since

lim pnj = p,
j→∞

pnj is a convergent subsequence of {pn }.


Suppose R is a infinite subset of Rk , since R is bounded, there exists a k-cell I such that

R = {pn | n ∈ N} is an infinite subset of the compact set I.


k
Bisecting sides of I into 2k k-cells {Qi }2i=1 with equal sizes such that

2k
[
I= Qi .
i=1

k
Let I1 ∈ {Qi }2i=1 such that
1
I1 ∩ R contains infinitely many elements and diam (I1 ) = diam (I).
2
Next subdivide I1 and continue the process to obtain a sequence of k-cells such that

I ⊃ I1 ⊃ I2 ⊃ · · ·
Ij ∩ R contains infinitely many elements for each j ∈ N
1 1
diam (Ij ) = diam (Ij−1 ) = · · · = j diam (I) → 0 as j → ∞
2 2
For each j ∈ N, since Ij contains infinitely many elements of {pn }, there is a subsequence

\
{nj } such that pnj ∈ Ij ∩ R. Next, since lim diam (Ij ) = 0, Ij contains exactly one point,
j→∞
j=1
say p ∈ Rk . This implies that

\
{p} = Ij and 0 ≤ lim d(pnj , p) ≤ lim diam (Ij ) = 0 =⇒ lim pnj = p.
j→∞ j→∞ j→∞
j=1

Theorem Let X be a metric space and let {pn } be a sequence in X. Suppose that E = {pn |
n ∈ N} is the set of points determined by {pn } and E ∗ is the set of all subsequential limits of
{pn } given by

E ∗ = {p ∈ X | ∃ a subsequence {pni } ⊂ {pn } such that lim pni = p}.


i→∞
0
Then E ∗ is closed, i.e. E ∗ ⊆ E ∗.

Page 4
Advanced Calculus Chapter 3 Summary(Continued) 2019-2020

Proof

Suppose E = {q1 , . . . , qm } contains finitely many points of X.


=⇒ E ∗ contains finite number of points
0
=⇒ E∗ = ∅ ⊆ E∗
=⇒ E ∗ is closed.

0
Suppose E contains infinitely many points of X and assume that E ∗ 6= ∅.
0
=⇒ Given q ∈ E ∗ , let pn1 ∈ {pn } such that pn1 6= q
=⇒ δ = d(q, pn1 ) > 0.

0
Since q ∈ E ∗
=⇒ ∃ x1 ∈ Nδ/2 (q) ∩ E ∗ \ {q} and since x1 ∈ E ∗
=⇒ ∃ n2 > n1 such that pn2 ∈ Nδ/2 (x1 ) ∩ E.
=⇒ d(q, pn2 ) ≤ d(q, x1 ) + d(x1 , pn2 ) < δ.

Similarly, ∃ x2 ∈ Nδ/22 (q) ∩ E ∗ \ {q} and since x2 ∈ E ∗


=⇒ ∃ n3 > n2 > n1 such that pn3 ∈ Nδ/22 (x2 ) ∩ E.
=⇒ d(q, pn3 ) ≤ d(q, x2 ) + d(x2 , pn3 ) < δ/2.

pn 1

δ δ/2

x1 pn2
δ/4
δ/4 q
x2
pn3
δ/2

For each k ≥ 2, suppose pn1 , . . . , pnk are chosen such that d(pnj , q) < δ/2j−2 for each 2 ≤ j ≤ k,
then we choose
xk ∈ Nδ/2k (q) ∩ E ∗ \ {q} and pnk+1 ∈ Nδ/2k (xk ) ∩ E

Page 5
Advanced Calculus Chapter 3 Summary(Continued) 2019-2020

such that
d(q, pnk+1 ) ≤ d(q, xk ) + d(xk , pnk+1 ) < δ/2k−1 .

δ/2k−2
pnk
δ/2k−1

δ/2k
q
xk
pnk+1
δ/2k

Since lim d(q, pnk ) ≤ lim δ/2k−2 = 0, {pnk } is a subsequence of {pn } converging to q and hence
k→∞ k→∞ 0 0
q ∈ E ∗ . Since q is an arbitrary point from q ∈ E ∗ , we have shown E ∗ ⊆ E ∗ and E ∗ is closed.

Cauchy Sequences
Question How to tell if {pn } converges if you don’t know the limit already?

Definition The sequence {pn } is a Cauchy sequence if for each  > 0, there exists N ∈ N such
that if m, n > N then it is implied that d(pm , pn ) < .

Definition Let E be a subset of a metric space X = (X, d). The diameter of E is defined by

diam E = sup{d(p, q) | p, q ∈ E}.

Remarks
(a) Note that 0 ≤ diam E ≤ ∞.
(b) Let {pn } be a sequence in X and let EN = {pn | n ≥ N }. Then {pn } is a Cauchy sequence
if and only if
lim diam EN = 0.
N →∞

(c) Let E be a subset of a metric space X = (X, d). Then diam Ē = diam E.
Proof Since E ⊆ Ē, we have diam E ≤ diam Ē.
Given  > 0 and for any p, q ∈ Ē, since Ē = E ∪ E 0 , there exist p0 , q 0 ∈ E such that
 
d(p, p0 ) < and d(q, q 0 ) < .
2 2

Page 6
Advanced Calculus Chapter 3 Summary(Continued) 2019-2020

This implies that


 
d(p, q) ≤ d(p, p0 ) + d(p0 , q 0 ) + d(q, q 0 ) ≤ + diam E + = diam E +  ∀ p, q ∈ Ē,
2 2
and
diam Ē ≤ diam E +  =⇒ diam Ē ≤ diam E by letting  → 0+ .
Hence we have diam Ē = diam E.
(d) If Kn is a sequence of compact sets in X such that Kn ⊃ Kn+1 for all n ∈ N and if

lim diam Kn = 0,
n→∞


\
then Kn consists of exactly one point.
n=1

Theorem
(a) In any metric space X, every convergent sequence is a Cauchy sequence, i.e. if {pn } ⊂ X
converges, then it is Cauchy.

Proof Suppose that lim pn = p. Then ∀  > 0 there exists N such that n > N d(pn , p) < .
n→∞ 2
So for m, n > N we know

d(pn , pm ) ≤ d(pn , p) + d(pm , p)


 
< +
2 2
= .

Remark Not every Cauchy sequence converges. To see this, let X = Q. Let pn be the
m
smallest such that > π. Then {pn } is Cauchy, but does not converge in Q (in R converge
n
to π).
(b) If X is a compact metric space and if {pn } is a Cauchy sequence in X, then {pn } converges
to some point of X.
Proof Consider the set S defined by

S = {pn | n ∈ N}, i.e. the range set of the map p : N → X defined by p(n) = pn .

Case 1 S contains finitely many points, say S = {pn1 , . . . , pnm }.


Let
δ = min{d(pni , pnj ) | 1 ≤ i < j ≤ m}.
Given  > 0 such that  < δ, since {pk } is Cauchy, there exists N ∈ N such that

if k, l ≥ N then d(pk , pl ) <  =⇒ if k, l ≥ N then d(pk , pl ) < δ.

Suppose ∃ k, l ≥ N such that pk 6= pl , then we must have

δ ≤ d(pk , pl ) < δ =⇒ δ < δ which is a contradiction.

Hence we have

∀ k, l ≥ N, pk = pl =⇒ if k ≥ N, then pk = pnj for some 1 ≤ j ≤ m.

Page 7
Advanced Calculus Chapter 3 Summary(Continued) 2019-2020

Therefore we have
lim pk = pnj for some 1 ≤ j ≤ m,
k→∞

i.e. {pk } converges to pnj ∈ X.


Case 2 S contains infinitely many points.
Since X is compact and S is an infinite subset of X,

X is closed, S 0 6= ∅ and S 0 ⊂ X 0 = X =⇒ ∃ p ∈ S 0 ⊂ X.

This implies that for each k ∈ N, there exists pnk ∈ B1/k (p) ∩ S \ {p}. Thus we obtain a
subsequence {pnk } of {pn } converging to p. This imples that

∀  > 0, ∃ N1 ∈ N such that if k ≥ N1 , then d(pnk , p) < .

Also since {pn } is a Cauchy sequence,

with the given  > 0, ∃ N2 ∈ N such that if n, m ≥ N2 , then d(pn , pm ) < .

Let N = max{N1 , N2 }. Since

d(pn , p) ≤ d(pn , pnn ) + d(pnn , p) < 2 ∀ n ≥ N =⇒ lim pn = p,


n→∞

i.e. {pn } converges to p ∈ X.


(c) In Rk , every Cauchy sequence converges.

Definition A metric space X is complete if every Cauchy sequence converges to a point in X.

Remarks
(a) Compact metric spaces are complete.
Proof Let {xi } be Cauchy in X. Then {xi } has a convergent subsequence. So there exist
{xnk } converging to x ∈ X. Fix  > 0. Cauchy implies there exists N ∈ N such that
i, j > N then d(xi , xj ) < 2 . By convergence of {xnk }, there exists N 0 ∈ N such that
whenever nk > N 0 , then d(x, xnk ) < 2 . Let N 00 = max (N, N 0 ). Then i, nk > N 00 implies

d(x, xi ) ≤ d(x, xnk ) + d(xnk , xi )


 
< +
2 2
= .

Hence the sequence itself converges to x and therefore X is complete.


(b) Rk is complete but Q is not.
(c) Let X be the ordered field R. Then X has the leat-upper-bound property if and only if
every Cauchy sequence converges to a point in X. Thus an ordered field X is complete if it
has the least-upper-bound property.
Proof
(=⇒) Let S be a nonempty, bounded from above subset of X = R. Choose a sufficiently
large M > 0 such that

M is an upper bound of S and [−M, M ] ∩ S 6= ∅.

Page 8
Advanced Calculus Chapter 3 Summary(Continued) 2019-2020

Let I1 = [−M, M ]. Divide I1 into 2 equal length subintervals and let I2 be the closed
subinterval such that

the right endpoint of I2 is an upper bound of S and I2 ∩ S 6= ∅.

Continuing this process, we obtain a sequence of closed intervals {In } such that

the right endpoint of In is an upper bound of S, In ∩ S 6= ∅ ∀ n ≥ 1.

and
M
I1 ⊃ I2 ⊃ · · · ⊃ In ⊃ · · · ; lim |In | = lim
= 0,
n→∞ n→∞ 2n−2

For each n ∈ N, since In ∩ S =


6 ∅, let xn be a point in In ∩ S.
M
Since lim |In | = lim n−2 = 0, {xn } is a Cauchy sequence and hence it converges to a
n→∞ n→∞ 2
point, say x, in X = R.

(⇐=) Let {xn } be a Cauchy sequence in X = R. Since {xn } is bounded, there exists
x ∈ X = R and a closed interval I = [−M, M ] such that

x = sup{xn | n ∈ N} and {xn } ⊂ I.

Since I is compact and {xn } is a Cauchy sequence, we have

lim xn = x.
n→∞

Theorem Every metric space (X, d) has a completion (X ? , ∆). In other words, (X ? , ∆) is a
complete metric space containing in X.
Let X ? = {Cauchy sequences in X}/ ∼. We will say:

{pn } ∼ {p0n } ⇐⇒ lim d(pn , p0n ) = 0.


n→∞

Let P, Q ∈ X ? . Then P = [{pn }], and Q = [{p0n }]. We define:

∆(P, Q) = lim d(pn , qn ).


n→∞

We claim (X ? , ∆) is a complete metric space and (X, d) is isometrically embedded in (X ? , ∆).


In other words, there is an injection i : X ,→ X ? such that d(p, q) = ∆(i(p), i(q)).

Remarks
(a) This is the other construction of R. But Dedekind cuts are more hardcore.
(b) Example If X = Q, then X ? = R. In particular, X ? is isometrically isomorphic to R. In
other there is a distance preserving bijection.

Definition A sequence {sn } in R is monotonically increasing if sn ≤ sn+1 for all n. Similarly


{sn } is monotonically decreasing if sn ≥ sn+1 .

Page 9
Advanced Calculus Chapter 3 Summary(Continued) 2019-2020

Theorem Suppose {sn } is a monotonic sequence of real numbers. Then {sn } converges if and
only if it is bounded.
Proof
(⇐=) Suppose sn ≤ sn+1 (the proof is analogous in the other case).
Let
E = {sn | n ∈ N} = the range of {sn }.
If {sn } is bounded, let
s = sup E = sup{sn | n ∈ N}.
Then
sn ≤ s ∀ n ∈ N.
For every  > 0, since s −  < s and there is an integer N such that

s −  < sN ≤ s,

for otherwise s −  would be an least-upper-bound of E. Since sn+1 ≥ sn for all n ∈ N, therefore


implies
if n ≥ N, then s −  < sN ≤ sn ≤,
which shows that {sn } converges to s.
(=⇒) Suppose that s = lim sn . By taking  = 1, there exists K ∈ N such that
n→∞

if n ≥ K, then |sn | − |s| ≤ |sn − n| <  = 1 =⇒ |sn | < 1 + |s| ∀n ≥ K.

Thus
if L = max{|s1 |, . . . , |sK−1 |, 1 + |s|} =⇒ |sn | ≤ L ∀ n ∈ N.
This proves that {sn } is bounded.

Remarks
(a) Monotone Sequence Property Let F be an ordered field. We say F has the monotone
sequence property if every bounded above monotonically increasing sequence converges.
(b) Let X be the ordered field R. Then X has the least-upper-bound property if and only if
X has the monotone sequence property. Thus an ordered field X is complete if it has the
monotone sequence property.
Proof
(⇐=) Let S be a nonempty, bounded from above subset of X = R. Choose a sufficiently
large M > 0 such that

M is an upper bound of S and [−M, M ] ∩ S 6= ∅.

Let I1 = [−M, M ]. Divide I1 into 2 equal length subintervals and let I2 be the closed
subinterval such that

the right endpoint of I2 is an upper bound of S and I2 ∩ S 6= ∅.

Continuing this process, we obtain a sequence of closed intervals {In } such that

the right endpoint of In is an upper bound of S, In ∩ S 6= ∅ ∀ n ≥ 1.

Page 10
Advanced Calculus Chapter 3 Summary(Continued) 2019-2020

and
M
I1 ⊃ I2 ⊃ · · · ⊃ In ⊃ · · · ; lim |In | = lim
= 0,
n→∞ n→∞ 2n−2

For each n ∈ N, by letting xn be the left endpoint of In , we obtain a bounded above


monotonically increasing sequence {xn }. Hence there exists x ∈ X = R such that

lim xn = x.
n→∞

M
Since lim |In | = lim = 0 and, for each n ≥ 1, the right endpoint of In is an upper
n→∞ n→∞ 2n−2
bound of S, x = sup{xn | n ∈ N} ∈ X = R.
(=⇒) Let {xn } be a bounded above monotonically increasing sequence in X = R. Since
{xn } is bounded above monotonically increasing, there exists x ∈ X = R such that

lim xn = x.
n→∞

Upper and Lower Limits

Definition Let {sn } be a sequence of real numbers.


(a) lim sn = ∞ iff ∀ M ∈ R, ∃ N ∈ N such that if n ≥ N then sn ≥ M.
n→∞
(b) lim sn = −∞ iff ∀ M ∈ R, ∃ N ∈ N such that if n ≥ N then sn ≤ M.
n→∞

Definition 3.16 Let {sn } be a sequence of real numbers and let E be the set of subsequential
limits of {sn } defined by
n o
E = x ∈ (−∞, ∞) ∪ {±∞} | ∃ {snk } ⊂ {sn } such that lim snk = x .
k→∞

Let

lim sup sn = s∗ = sup E,


n→∞
lim inf sn = s∗ = inf E.
n→∞

The numbers s∗ , s∗ are called the upper and lower limits of {sn }; we use the notation

lim sup sn = s∗ , lim inf sn = s∗ .


n→∞ n→∞

Theorem 3.17 Let {sn } be a sequence of real numbers and let E be the set of subsequential
limits of {sn }. Then s∗ = sup E and s∗ = inf E have the following properties:
(a) s∗ , s∗ ∈ E, i.e. there exist subsequences {snk }, {smj } of {sn } such that

lim snk = s∗ , lim smj = s∗ .


k→∞ j→∞

Proof
If s∗ = ∞, then E is not bounded above; hence {sn } is not bounded above, and there is a
subsequence {snk } such that lim snk = ∞.
k→∞

Page 11
Advanced Calculus Chapter 3 Summary(Continued) 2019-2020

If s∗ ∈ R = (−∞, ∞), then E is bounded above; and at least one subsequential limit exists,
so that (a) follows from the Weierstrass Theorem.
If s∗ = −∞, then E contains only one element, namely −∞, and there is no subsequential
limit. Hence for any M ∈ R, there exist at most a finite number of values of n such that
sn > M which implies that lim = −∞.
n→∞

(b) If x > s ≥ s∗ > y, there exists an N ∈ N such that n ≥ N implies x > sn > y, i.e. for each
 > 0, there exists an N ∈ N such that
if n ≥ N then s∗ −  < sn < s∗ + .

s∗ −  s∗ + 

Proof For each  > 0, if there are infinitely many sn ’s lying in (−∞, s∗ − ] ∪ [s∗ + , ∞),
then there exists a subsequence {snk } lying completely in (−∞, s∗ − ] or [s∗ + , ∞).
This implies that there exists a subsequential limit x ∈ (−∞, s∗ − ] ∪ [s∗ + , ∞) ∪ {±∞}
which is a contradiction to the definition of s∗ = inf E or that of s∗ = sup E.
Hence there are only finite number of xn ’s lying in (−∞, s∗ − ] ∪ [s∗ + , ∞).

Remarks
(a) If k ≥ l ∈ N, then {sm | m ≥ k} ⊆ {sm | m ≥ l} and
inf{sm | m ≥ l} ≤ inf{sm | m ≥ k} ≤ sup{sm | m ≥ k} ≤ sup{sm | m ≥ l},
i.e. sup{sm | m ≥ k} is monotone decreasing in k, and inf{sm | m ≥ k} is monotone
increasing in k.
(b) If s∗ , s∗ ∈ E, and {snk }, {smj } are subsequences of {sn } such that
lim snk = s∗ , lim smj = s∗ =⇒ lim inf snk = s∗ = lim sup snk , lim inf smj = s∗ = lim sup smj .
k→∞ j→∞ k→∞ k→∞ j→∞ j→∞

For each  > 0, since there only exist finite number of n ∈ N such that sn > s∗ +  or
sn < s∗ − , hence we have
s∗ −  ≤ inf{sm | m ≥ n} ≤ sup{sm | m ≥ n} ≤ s∗ +  for sufficiently large n
=⇒ s∗ −  ≤ lim inf{sm | m ≥ n} ≤ lim sup{sm | m ≥ n} ≤ s∗ +  ∀  > 0
n→∞ n→∞
=⇒ lim+ (s∗ − ) ≤ lim inf{sm | m ≥ n} ≤ lim sup{sm | m ≥ n} ≤ lim+ (s∗ + )
→0 n→∞ n→∞ →0

=⇒ s∗ ≤ lim inf{sm | m ≥ n} ≤ lim sup{sm | m ≥ n} ≤ s .
n→∞ n→∞

On the other hand for each l ∈ N, since {smj | j ≥ l} and {snk | k ≥ l} are subsequences of
{sn | n ≥ ml } and {sm | m ≥ nl } respectively, we have
inf{sn | n ≥ ml } ≤ smj and snk ≤ sup{sm | m ≥ nl } ∀ j, k ≥ l
=⇒ inf{sn | n ≥ ml } ≤ lim smj = s∗ and s∗ = lim snk ≤ sup{sm | m ≥ nl } ∀ l ∈ N
j→∞ k→∞

=⇒ inf{sn | n ≥ ml } ≤ s∗ ≤ s ≤ sup{sm | m ≥ nl } ∀ l ∈ N
=⇒ lim inf{sn | n ≥ ml } ≤ s∗ ≤ s∗ ≤ lim sup{sm | m ≥ nl }
l→∞ l→∞

=⇒ lim inf{sm | m ≥ n} ≤ s∗ ≤ s ≤ lim sup{sm | m ≥ n}.
n→∞ n→∞

Page 12
Advanced Calculus Chapter 3 Summary(Continued) 2019-2020

Hence

lim sup sn = s∗ = lim sup{sm | m ≥ n} = inf{sup{sm | m ≥ n} | n ∈ N},


n→∞ n→∞

lim inf sn = s∗ = lim inf{sm | m ≥ n} = sup{inf{sm | m ≥ n} | n ∈ N}.


n→∞ n→∞

(c) If x > s∗ = lim sup{sm | m ≥ n} and lim inf{sm | m ≥ n} = s∗ > y, there exists an
n→∞ n→∞
N ∈ N depending on x − s∗ > 0 and s∗ − y > 0 such that if n ≥ N then

max{| sup{sm | m ≥ n} − s∗ |, |s∗ − inf{sm | m ≥ n}|} < min{x − s∗ , s∗ − y} ∀ n ≥ N


=⇒ sup{sm | m ≥ n} − s∗ < x − s∗ and s∗ − inf{sm | m ≥ n} < s∗ − y ∀ n ≥ N
=⇒ sn ≤ sup{sm | m ≥ n} < x and sn ≥ inf{sm | m ≥ n} > y ∀ n ≥ N
=⇒ y < inf{sm | m ≥ n} ≤ sn ≤ sup{sm | m ≥ n} < x ∀ n ≥ N.

Examples
(a) Let {sn } = Q, be a sequence containing all rationals. Then every real number is a subse-
quential limit, and
lim sup sn = ∞, lim inf sn = −∞,
n→∞ n→∞

i.e. the set of subsequential limits of {sn } is E = R ∪ {±∞}.


(−1)n
(b) Let sn = . Then
1 + (1/n)

lim sup sn = 1, lim inf sn = −1.


n→∞ n→∞

(c) For a real-valued sequence {sn },

lim sn = s ⇐⇒ lim sup sn = lim inf sn = s.


n→∞ n→∞ n→∞

Theorem If sn ≤ tn for n ≥ N, where N is fixed, then

lim inf sn ≤ lim inf tn , lim sup sn ≤ lim sup tn .


n→∞ n→∞ n→∞ n→∞

Some Special Sequences

Theorem
1
(a) If p > 0, then lim = 0.
n→∞ np
Proof For each  > 0, there exists n ∈ N such that
1 1 1 1
n> ⇐⇒ np > ⇐⇒  > p = | p − 0|.
1/p  n n

Page 13
Advanced Calculus Chapter 3 Summary(Continued) 2019-2020

(b) p > 0, then lim n
p = 1.
n→∞

Proof If p > 1, by letting xn = n
p − 1 > 0, then
p−1 √
1 + n xn ≤ (1 + xn )n = p =⇒ xn ≤ =⇒ lim xn = 0 ⇐⇒ lim n p = 1.
n n→∞ n→∞

1
If p < 1, by letting p = for some q > 1, we have
q
√ 1 √ 1
lim n
q = 1 =⇒ lim √ = 1 =⇒ lim n p = lim √ = 1.
n→∞ n→∞ n q n→∞ n→∞ n q


n
(c) lim n = 1.
n→∞

Proof Let xn = n
n − 1. Then xn ≥ 0 for each n ∈ N and
r
n n(n − 1) 2 2
n = (1 + xn ) ≥ xn =⇒ 0 ≤ xn ≤ for n ≥ 2 =⇒ lim xn = 0.
2 n−1 n→∞


(d) If p > 0 and α ∈ R, then lim = 0.
n→∞ (1 + p)n
n
Proof Let k ∈ N such that k > α. If n ∈ N and > k, then
2
n k n(n − 1) · · · (n − k + 1) k nk pk nα 2k k! α−k
 
n
(1 + p) > p = p > k =⇒ 0 < < n .
k k! 2 k! (1 + p)n pk

(e) If |x| < 1, then lim xn = 0.


n→∞
1
Proof Take α = 0 and p > 0 such that |x| = as in (d).
1+p

Series
Question What does this mean?
1 1 1 1 π2
1+ + + + + ··· = .
4 9 16 25 6

Remark Sum of natural numbers to negative even powers always has a nice form.
Consider also:
1 − 1 + 1 − 1 + 1 − 1 + ...
You can associate these differently and get different limits. This tells us we cannot assume
associativity in infinite sums.

Notation Let {an } be a real sequence. Then:


j
X
an = ai + ai+1 + · · · + aj ,
n=i

when i < j ∈ N.

Page 14
Advanced Calculus Chapter 3 Summary(Continued) 2019-2020

Definition The nth partial sum of {ak } is


n
X
sn = ak .
k=1

Remark {sn } is a real sequence. Sometimes {sn } is called an infinite series.

Definition If sn → s we write

X n
X
an = lim ak = s
n→∞
n=1 k=1

Question When does an infinite series converge? When its sequence of partial sums converge.


X 1
Example Does converge?
n=1
n
n
X 1
Consider partial sums where sn = . We use the Cauchy criterion. For m < n, then
k=1
k

d(sm , sn ) = sn − sm
= sm+1 + sm+2 + · · · + sn
1 1 1 n−m
= + + ··· + ≥ .
m+1 m+2 n n
1
The inequality comes from observing that all the terms in the sum are less than or equal to .
n
2n − n 1
Thus s2n − sn ≥ = . Therefore this sequence is not Cauchy. Hence {sn } does not
2n 2
converge.

X
Cauchy Criterion for series an converges if and only if for all  > 0 there exists N ∈ N
such that m ≥ n > N implies
m
X


a k
< .

k=n
X
Corollary (Divergence Test) If an converges, then lim an = 0.
n→∞

Remarks
X1
(a) is called the harmonic series.
n
(b) The corollary’s converse is not true (counter-example is harmonic series).

X
Theorem If an ≥ 0, then an converges if and only if partial sums {sn } form a bounded
sequence.
Proof
X Since ak ≥ 0 for all k ∈ N, {sn } is monotonically increasing. Thus {sn } converges, i.e.
an converges, if and only if {sn } is bounded.

Page 15
Advanced Calculus Chapter 3 Summary(Continued) 2019-2020

Comparison Test
X X
1. If cn converges and |an | ≤ cn for almost all n, then an converges.
X X
2. If dn diverges to +∞ and an ≥ dn for almost all n, then an diverges as well.

Proof

X
1. Let  > 0. Since cn converges, it satisfies Cauchy Criterion. Thus there exists N ∈ N
such that m ≥ n ≥ N implies:
m
m
X X
ck ≤ ck < .
k=n k=n

Thus m m m
X X X

a k
≤ |a k | ≤ ck .
k=n k=n k=n
This follows from the assumption that |an | ≤ cn for almost all n so let N be at least larger
X n for which cn < |an |. The resulting inequality satisfies the Cauchy Criterion
than the last
and thus an converges.
2. Follows from (a) via contrapositive. (Also, partial sums form a bounded sequence.)

Geometric Series If |x| < 1, then



X 1
xn = .
n=0
1−x
X
n
If |x| ≥ 1, then x diverges.
n
X 1 − xn+1
Proof If x 6= 1, let sn = xk = 1 + x + · · · + xn . Then sn = by multiplying both
k=0
1−x
sides by 1 − x. Thus it follows:
1
lim sn = if |x| < 1
n→∞ 1−x
If |x| > 1, then {sn } does not converge. Similarly if x = ±1, use the divergence test to verify
{sn } does not converge.


X 1
Example converges. Notice
n=0
n!
1 1 1 1
sn = 1 + 1 + + + + ··· +
2! 3! 4! n!
1 1 1 1
≤ 1 + 1 + + 2 + 3 + · · · + n−1
2 2 2 2
< 3.
Thus it is bounded and since each term is nonnegative, it is monotonically increasing. Thus it
converges.

Page 16
Advanced Calculus Chapter 3 Summary(Continued) 2019-2020


X 1
Definition e = .
n=0
n!

1 n
Theorem e = lim (1 + ) .
n→∞ n
Proof n
X 1 1 n
Let sn = , tn = 1 + .
k=0
k! n
By the binomial theorem,
1 1 1 1 2
tn = 1 + 1 + 1− + 1− 1− + ···
2! n 3! n n
1 1 2 n − 1
+ 1− 1− ··· 1 − .
n! n n n
Hence tn ≤ sn ≤ e for all n ∈ N, so that

lim sup tn ≤ e.
n→∞

Next, if n ≥ m, then
1 1 1 1 2 m − 1
tn ≥ 1 + 1 + 1− + ··· + 1− 1− ··· 1 − .
2! n m! n n n
By keeping m fixed and letting n → ∞ on both sides, we get
1 1
lim inf tn ≥ 1 + 1 + + ··· + =⇒ lim inf tn ≥ sm ∀ m ∈ N.
n→∞ 2! m! n→∞

Letting m → ∞, we finally get


1 n
e ≤ lim inf tn ≤ lim sup tn ≤ e =⇒ e = lim tn = lim (1 + ) .
n→∞ n→∞ n→∞ n→∞ n

Remark
1 1
e − sn = + + ...
(n + 1)! (n + 2)!
1 1 1
< (1 + + + ...)
(n + 1)! n + 1 (n + 1)2
 
1 1 1
= =
(n + 1)! 1 − 1/(n + 1) n! n

Theorem e 6∈ Q.
m
Proof Suppose e = for m, n > 0. Then
n
1
0 < n! (e − sn ) < < 1
| {z } n
∈N

Page 17
Advanced Calculus Chapter 3 Summary(Continued) 2019-2020

Recall A series converges if its partial sums converges.

1 1 1 1
Example 1 + + + + + . . . converges because partial sums converge.
2 4 8 16

X
Cauchy’s Theorem If a1 ≥ a2 ≥ a2 · · · ≥ 0, i.e. monotonically decreasing, then an con-
n=1

X
verges if and only if 2k a2k = a1 + 2a2 + 4a4 + 8a8 + . . . converges.
k=0

Proof Compare sn = a1 + · · · + an and tk = a1 + 2a2 + · · · + 2k a2k . Consider the following


grouping of the finite sum:

sn = (a1 ) + (a2 + a3 ) + (a4 + · · · + a7 ) + · · · + an


tk = (a1 ) + (a2 + a2 ) + (a4 + · · · + a4 ) + · · · + (a2k + · · · + a2k )

If tk converges, since sn < tk ∀ n < 2k and ∀ k ∈ N,


=⇒ sn ≤ lim tk ∀ n ∈ N
k→∞
=⇒ sn converges since sn is monotonically increasing and bounded above.

On the other hand,

if sn converges, since 2sn > tk − a1 ∀ n ≥ 3 and ∀ 2k < n,


=⇒ 2 lim sn ≥ tk − a1 ∀ k ∈ N
n→∞
=⇒ tk converges since tk − a1 is monotonically increasing and bounded above.

This proves that both series diverge or converge simultaneously.

P 1
Theorem Consider . Claim is that this converges if p > 1 and diverges if p ≤ 1.
np
Proof If p ≤ 0, terms do not go to zero, so the series diverges. If p > 0, look at:
X 1 X
2k = 2(1−p)k ,
k
(2k )p k

which is geometric. Thus it converges if and only if 21−p < 1. This only happens when 1 − p < 0
and hence p > 1.

Remark We were able to turn a harmonic-like series into a geometric-like series.

P p
Theorem (Root Test) Given an , let α = lim n |an |. Then
n→∞
P
(a) if α < 1, then an converges;
P
(b) if α > 1, then an diverges;
(c) if α = 1, then the test is inconclusive.

Page 18
Advanced Calculus Chapter 3 Summary(Continued) 2019-2020

Proof If α < 1, we can choose β so that α < β < 1, and an integer N such that
p
n
|an | < β for n ≥ N, by Theorem 3.17(b).

That is, if n ≥ N, then |an | < β n . Since 0 < β < 1,


P n P
β converges. Therefore, an converges
(absolutely) by the comparison test.
If α > 1, then, again by Theorem 3.17(b), there exists a sequence {nk } such that
q
nk
|ank | → α > 1.

Hence |an | > 1 for infinitely many values of n, and


X
lim an 6= 0 =⇒ an diverges.
n→∞

P1 P 1
Since diverges, converges and α = 1 in both cases, the test is inconclusive if α = 1.
n n2
P
Theorem (Ratio Test) The series an
an+1
(a) converges if lim sup < 1,
an
an+1
(b) diverges if ≥ 1 for n ≥ n0 , where n0 is some fixed integer.
an
an+1
Proof If lim sup < 1, there exist β < 1 and an integer N such that
an
an+1
< β < 1 for n ≥ N.
an
This implies that

|aN +k | < β|aN +k−1 | < β 2 |aN +k−2 | < · · · < β k |an | for each k ∈ N.
P
Therefore, an converges (absolutely) by the comparison test.
If |an+1 | ≥ |an | for n ≥ n0 , then
X
lim an 6= 0 =⇒ an diverges.
n→∞


X
Definition A power series is of the form cn z n where cn ∈ C.
k=0

p
Theorem Let α = lim sup n |cn |. Let r = α1 . Then the power series converges if |z| < R and
diverges |z| > R. We call R the radius of convergence.
p p p
Proof Use the root test so consider lim sup n |cn z n | = lim sup |z| n |cn | = |z| lim sup n |cn |.
n→∞ n→∞ n→∞
Notice that this less than 1, and thus converges, when |z| is less than 1 over the limsup.

P
Definition A series converges absolutely if |an | converges.

Page 19
Advanced Calculus Chapter 3 Summary(Continued) 2019-2020

P P
Theorem an converges absolutely implies an converges.
m
X m
X P
Proof | ak | ≤ |ak | < , by the Cauchy Criterion since |ak | converges.
k=n k=n

Example If a series converges, it does not necessarily converge absolutely. Consider the series
1 − 21 + 13 − 14 + . . . , which converges by alternating series test but does not converge absolutely.

Question If the terms in a convergent series are rearranged, must it converge to same sum? Not
all the time, but it does if the series converges absolutely.

P
Riemann’s Theorem If a series an converges but not absolutely, then we can form a rear-
rangement that has any limsup and liminf you’d like.

Example Rearrange 1 − 12 + 31 − 14 + . . . to converge to 4. We want to the partial sums to


converge to 4. Consider the positive terms in sequence that sum to at least 4 (notice the positive
terms diverge). Then use a s many negative terms you need to go back, as many positive terms
you need to go forward, et cetera. These partial sums converge to 4.

Page 20

You might also like